Bất đẳng thức AM-GM

V) Bất đẳng thức holder

Bất đẳng thức Holder là một bất đẳng thức khá mạnh và có nhiều ứng dụng,

nhưng rất tiếc nó không được phổ biến ở phổ thông hiện nay. Đặc biệt, bất đẳng thức

holder còn được coi là dạng tổng quát của bất đẳng thức AM-GM (với m = 2 ) và cách

chứng minh bất đẳng thức này có sử dụng đến bất đẳng thức AM-GM. Vì thế, chúng tôi

xin giới thiệu bất đẳng thức này trong chuyên đề.

pdf31 trang | Chia sẻ: tuanbinh | Lượt xem: 993 | Lượt tải: 0download
Bạn đang xem trước 20 trang tài liệu Bất đẳng thức AM-GM, để xem tài liệu hoàn chỉnh bạn hãy click vào nút TẢi VỀ
đạt được khi . 
 Như vậy việc đưa số vào áp dụng BĐT là hoàn toàn có cơ sở. Từ đó ta nâng bài 
toán lên với hệ số các số hạng là các số dương như sau: 
Ví dụ 2: Cho . 
 Tìm GTNN của 
 7 
 Lời giải: 
 Mục tiêu của chúng ta là dùng bất đẳng thức AM-GM sao cho khi cộng 2 bất đẳng 
thức vào, ta có vế trái là 2F cộng với 1 số hạng nào đó, còn vế phải chứa biểu thức đã cho 
trong giả thiết. Rõ ràng việc đặt số đơn lẻ sẽ không đưa đến kết quả mà phải biến đổi 
số hạng cộng vào mỗi bất đẳng thức. 
 Cách đặt số hạng cộng vào này giúp ta triệt tiêu được c bên vế trái, nhân thêm 
được hệ số a vào vế phải. Ta tiếp tục cộng 2 bất đẳng thức : 
 Dấu "=" xảy ra khi và chỉ khi 
 . 
 Khi đó . Giả sử đã có  thỏa mãn dấu "=", tức là: 
 (1) 
 Khi đó theo (1) tìm được GTNN của F là 
 Lần này, ta sẽ phát triển bài toán theo hướng tăng dần số mũ. Để tránh phức tạp, ta 
cho các hệ số bằng 1. 
 Ví dụ 3: Cho . Tìm GTNN của 
 Lời giải 
 Áp dụng bất đẳng thức AM-GM cho 4 số dương: 
 ; 
 8 
 Ở đây, ta cộng 3 số hạng bậc 4 của x với 1 số hạng tự do. Mục đích là để khi ta áp 
dụng BĐT AM-GM, ta thu được một số hạng bậc 3 của x. 
 Cộng 2 bất đẳng thức, ta được : 
 . 
 Dấu "=" xảy ra khi và chỉ khi: . 
 Khi đó (2). Giả sử tồn tại để dấu bằng xảy ra, vậy thì: 
 . 
 Thay vào (2) ta có , đạt được khi x = y = . 
 Qua các ví dụ trên, có lẽ các bạn đã phần nào hiểu được kĩ thuật này. Tuy nhiên, 
các ví dụ này vẫn còn rất đơn giản và mang tính chất tương tự nhau. Chúng ta hãy xét một 
trường hợp khác với các ví dụ trên xem thế nào? 
Ví dụ 4. Chứng minh rằng nếu x, y, z là các số thực không âm thì 
      
3
2 2 2 2 2 2 26 27 10x y z x y z xyz x y z         
 Lời giải: 
 Sử dung nguyên lý cơ bản “dấu bằng xảy ra khi một cặp biến số nào đó bằng 
nhau”, ta có thể tìm được dấu bằng của bất đẳng thức trên xảy ra khi y = z = 2x. Điều này 
cho phép chúng ta mạnh dạn đánh giá như sau: 
     
3
2 2 2 2 2 2210 6VP VT x y z x y z x y z          
      
1
2 2 2 2 2 2 210 6x y z x y z x y z
 
         
 
        
1 1
2 2 2 2 2 2 2 2 22 2
10
. 1 2 2 6
3
x y z x y z x y z
 
           
 
      2 2 2 10 . 2 2 63x y z x y z x y z
           
 9 
  2 2 2 28 2 2
3
x y z x y z   
 (1) 
 Áp dụng bất đẳng thức AM-GM, ta có: 
4 42 2 2 2 2 8 8
2 2 2 2 2 99
84 4 9 94 4 4 4 4
y z y z x y z
x y z x x
       
             
       
8 7928 2 2 7.4 2 2 9 4x y z x y z x yz      
 Nhân hai bất đẳng thức trên vế theo vế, ta được: 
   
2 8 8
2 2 2 8 79 9
8
28 2 2 9 4 .9 81
4
x y z
x y z x y z x yz xyz      (2) 
 Từ (1) và (2), ta suy ra bất đẳng thức cần chứng minh. 
 Trong ví dụ này, chúng ta đã sử dụng bất đẳng thức AM-GM cùng với kĩ thuật 
chọn điểm rơi. Lời giải rất hiệu quả và ấn tượng. Tuy nhiên sự thành công của lời giải trên 
nằm ở ba dòng ngắn ngủi ở đầu. Không có được “dự đoán” đó, khó có thể thu được kết 
quả mong muốn. 
 Chúng ta hãy xét tiếp một ví dụ khác: 
Ví dụ 5. Cho a;b;c>0. Chứng minh rằng : 
     
3 3 3
3 3 33 3 3
1
a b c
a b c b a c c b a
  
     
 Ý tưởng: Đây là một bất đẳng thức hay và khó.Chúng ta có khá nhiều cách giải bài 
này, ví dụ như : 
 Sử dụng bất đẳng thức AM-GM, ta có: 
       
2 2
3 2
1 1
1 1 1 1
2 2
x x x x
x x x x
   
        
 Vì vậy, ta có: 
 
3
3 2 2 23 3
2
1 1 1
1
1 111 2
cyc cyc cyc cyc
a
c ba b c b cb c
aaa
   
           
    
 10 
 Tuy nhiên, ở đây, chúng tôi xin giới thiệu 1 cách giải khác khá độc đáo và có sử 
dụng kĩ thuật chọn điểm rơi như sau: 
 Nhận xét rằng dấu bằng xảy ra khi và chỉ khi a = b = c. 
 Ta sẽ tìm p sao cho bất đẳng thức sau đúng: 
 
3
33
p
p p p
a a
a b ca b c

  
 Thay a = b = c vào bất đẳng thức trên, ta suy ra được p = 2. 
 Vậy, vấn đề của ta bây giờ là chứng minh bất đẳng thức sau đúng. 
  
3 2
3 2 2 23
a a
a b ca b c

  
 Từ những dự đoán và suy luận như trên, ta có lời giải bài toán trên như sau: 
 Lời giải: 
Ta sẽ chứng minh :  
3 2
3 2 2 23
a a
a b ca b c

  
 (*) 
 Thật vậy: 
 
2 2 233
1
(*)
a
a b ca b c
 
  
     2 32 2 2 3a b c a a b c      
      2 32 2 2 2 22a b c b c a b c      
         32 2 2 2 2 2b c a b a c a b c       
             2 2 32 2 2 22b c a b a c b c a b c a b c          
           2 2 22 2 0b c a b a c a b c b c         (Đúng) 
 Vậy (*) đúng. Chứng minh tương tự, ta có: 
  
3 2
3 2 2 23
b b
a b cb a c

  
 11 
  
3 2
3 2 2 23
c c
a b cc a b

  
 Do đó 
     
3 3 3 2 2 2
3 3 3 2 2 23 3 3
1
a b c a b c
a b ca b c b a c c b a
 
   
      
 Vậy, ta suy ra bất đẳng thức cần chứng minh. 
Sau đây là một số bài tập nho nhỏ về kĩ thuật này: 
1) cho 10;
2
a    
 Tìm min 
2
1
a
a
 
2) cho  ;a m n 0m n  ;m n . Tìm min 1n ma a
3) Cho 
3
; ;
4
a b c   1a b c   
Chứng minh: 2 2 2
9
1 1 1 10
a b c
a b c
  
  
4) Cho ; ;a b c   ; 3a b c   
 Chứng minh: 
a)
5 5 5 52 2 2 3 3a b b c c a      
b) 55 5 5( )(2 ) ( )(2 ) ( )(2 ) 3 6a a c a b b b a b c c c b c a         
5) cho ; ;a b c  thỏa mãn 3a b c   . CMR: 3 4 5 1a b c  
6) 0a b  CMR: 2
32
2 5
( )(2 3)
a
a b b
 
 
7) Cho 4a b  CMR: 
6 10
2 3 18a b
a b
    
8) Cho  ; ; 2;2a b c  và 3a b c   , CMR: 2 2 24 4 4 3 3a b c      
9) Cho ; ;x y z  CMR: 4
1
(1 3 )( 8 )( 9 )( 6) 7
xyz
x x y y z z

   
 12 
III) Kĩ thuật Cô-si ngược dấu: 
Các bạn cứ tưởng tượng rằng khi ta dùng bất đẳng thức AM-GM rồi nghịch đảo 
lại, rồi lại đặt dấu trừ ở trước thì hiển nhiên bất đẳng thức sẽ không đổi chiều. 
Sau đây là một số ví dụ để các bạn nắm rõ kĩ thuật này và một số bài để tự luyện. 
 Các bài toán mẫu: 
Bài toán 1. Cho các số dương a,b,c thỏa mãn a+b+c=3. Chứng minh 
Lời giải: 
Ta có : 
Tương tự cho 2 bất đẳng thức kia, ta suy ra: 
Bài toán 2. Cho các số thực dương a,b,c có tổng bằng 3. Chứng minh 
Lời giải: ta có: 
 Tương tự cho 2 bất đẳng thức còn lại ta suy ra 
Một số bài tập tự luyện: 
Bài 1. Cho các số dương a,b,c,d thỏa mãn a+b+c+d=4. Chứng minh 
 + 
Bài 2. Cho các số dương a,b,c,d thỏa mãn a+b+c+d=4. Chứng minh 
Bài 3 . Chứng minh rằng với mọi số thực dương ta luôn có: 
 13 
Bài 4. Chứng minh rằng với mọi số thực dương ta luôn có: 
Bài 5. Chứng minh rằng với mọi số thực không âm a,b,c 
sao cho a+b+c=3 thì 
Bài 6. Cho các số thực dương a,b,c có tổng bằng 4. Chứng minh 
Bài 7. Với mỗi số thực dương a,b,c,d có tổng bằng 4. 
 Chứng minh rằng : 
 Tóm lại kĩ thuật Cauchy ngược dấu rất có lợi trong các bất đẳng thức hoán vị 
IV) Bất đẳng thức Cauchy-Schwarz: 
Đã nhắc đến các bất đẳng thức cổ điển, đặc biệt là bất đẳng thức AM-GM, chúng 
không thể không nhắc đến bất đẳng thức Cauchy-Schwar. Đây là bất đẳng thức cũng rất 
quen thuộc với các bạn học sinh phổ thông và việc nắm chắc sử dụng thành thạo bất đẳng 
thức này là rất cần thiết cho tất cả bạn đọc, không chỉ với các bạn thi học sinh giỏi mà 
ngay cả với các bạn ôn thi vào Đại học. 
Trước hết xin nhắc lại về bất đẳng thức này, nó được phát biểu như sau: 
    22 2 2 2 2 21 2 1 2 1 1 2 2... ... . . ... .n n n na a a b b b a b a b a b          
Dấu “=” xảy ra khi và chỉ khi: i j j ia b a b với mọi i j 
Chứng minh: Có lẽ các bạn ai cũng biết cách chứng minh bất đẳng thức này nên 
chúng tôi chỉ xin nêu 1 cách chứng minh đơn giản và tiêu biểu sau : 
Xét tam thức bậc 2 sau đậy: 
       2 2 21 1 2 2 ... n nf x a x b a x b a x b       
Sau khi khai triển ta có: 
 14 
       2 2 2 2 2 2 21 2 1 1 2 2 1 2... 2 ... ...n n n nf x a a a x a b a b a b x b b b            
Mặt khác, vì   0f x x   nên theo định lý về tam thức bậc 2. 
    22 2 2 2 2 21 2 1 2 1 1 2 20 ... ... ...f n n n na a a b b b a b a b a b             
Đẳng thức xảy ra khi phương trình   0f x  có nghiệm, nói cách khác 
i j j ia b a b với mọi i j . 
Ta có 1 dạng khác của bất đẳng thức này và cũng rất hay được sử dụng và nó được 
phát biểu như sau: 
Với 2 bộ số  1 2; ;...; na a a và  1 2; ;...; nb b b thỏa mãn bi dương, ta có: 
 222 2 1 21 2
2 2 2
1 2 1 2
...
...
...
nn
n n
a a aaa a
b b b b b b
  
   
   
Dấu “=” cũng xảy ra khi và chỉ khi i j j ia b a b với mọi i j 
( Lưu ý là để sử dụng tốt dạng, các bạn cần có cái nhìn “ hai chiều ”) 
Sau đây là một số ví dụ cơ bản: 
Ví dụ 1. cho a;b;c là 3 số thực dương tùy ý. Chứng minh rằng: 
2 2 2 4
bc ac ab a b c
b c a a c b b a c
 
  
      
Lời giải: Sử dụng bất đẳng thức Cauchy-Schwarz, ta có: 
2 2 21 1 (1 1)
( ) ( )a b a c a b a c

 
     
Suy ra: 
1 1 1 1
2 4a b c a b a c
       
Theo đánh giá này, ta được: 
1
2 4 4
bc bc bc a b c
a b c a b a c
         
  
 15 
Đó chính là điều phải chứng minh. 
Nhận xét: Lời giải tuy đơn giản nhưng để tìm được hướng làm thì không dễ chút 
nào. Điểm đặc biệt ở đây là việc phát hiện ra hằng đẳng thức: 
bc bc
a b c
a b a c
       
 
Từ đó, chúng ta có cách tách thích hợp để có thể sử dụng bất đẳng thức Cauchy-
Schwarz như trên. 
Ví dụ 2. cho a, b, c là các số dương có tổng bằng 3. Chứng minh rằng: 
2 2 2 2 2 2 2 2 2
1 1 1 1
4 4 4 2a b c b a c c b a
  
      
Gợi ý lời giải: Sử dụng ý tưởng như trên, ta sẽ tìm một hằng đẳng thức thích hợp 
để có thể giúp ta chứng minh bất đẳng thức đã cho. Chúng ta hãy thử để ý đến hằng đẳng 
thức sau : 
2 2
2 2 2 2 3
b c
a b a c
 
    
 
Từ đó, ta nghĩ đến cách tách    2 2 2 2 2 2 2 24 2a b c a a b a c       và sử 
dụng bất đẳng thức Cauchy-Schwarz như sau: 
 2 2 2 2
2 2 2 2 2 2 2 24 2
a b c a b c
a b c a a b a c
 
  
    
Theo đánh giá này, ta có: 
2 2 2
2 2 2 2 2 2 2 2
1 9
4 2 2
a b c
a b c a a b a c
 
        
   
Chia cả 2 vế cho 9, ta được bất đẳng thức cần chứng minh. 
Ví dụ 3. Chứng minh rằng nếu a, b, c, d là các số thực dương và 4 1r abcd  , bất đẳng 
thức sau luôn đúng: 
 16 
 24 11 1 1 1
1 1 1 1 1
rab bc cd ad
a b c d r
   
   
    
Lời giải: Chú ý rằng giả thiết 4 1r abcd  ta suy ra tồn tại các số thực dương x; 
y; z; t sao cho: , , .
ry rz rt rx
a b c d
x y z t
    
Khi đó, bất đẳng thức được viết lại dưới dạng sau: 
 
22 22
211 11 4 1
11 1 1 1
r tr z r yr x
ryx tz
ry rx rt rx r
x y z t
  
   
   
 22 2 2 2 4 1
1
rr z x r t y r x z r y t
ry x rx y rt z rx t r
   
    
    
Xét 2 biểu thức 
x z y t x z y t
A
ry x rx y rt z rx t
   
   
    ;
z t x y
B
ry x rx y rt z rx t
   
   
Ta phải chứng minh 
    
2
2
4 1
1
1
r
A r B
r

  

Theo bất đẳng thức AM-GM dễ thấy 
   4 8r xy yz zt tx xz yt     
       4 1 4 8 8r x z y t x z y t xz yt         
        2 2 21 2 1r x y z t x y z t r x y z t              
 Sử dung bất đẳng thức Cauchy-Schwarz, chú ý rằng 1r  ta có: 
    
1 1 1 1
A x z y t
ry x rt z rx y rz t
   
              
 17 
     
      
2
2 2
4 4 4
2
x z y t x y z t
x z ry rt y t rx rz x z y t r x z y t
    
  
           
8
1r


 
       
2
x y z t
B
z xy x t rz y x rt z y rx t
  

      
 
   
2
4
2 1
x y z t
r xy yz zt tx xz yt r
  
 
     
 Từ 2 bất đẳng thức trên, ta có điều phải chứng minh. 
Đẳng thức xảy ra khi và chỉ khi a = b = c= d = r. 
Ví dụ 4. Cho a, b, c > 0 thỏa mãn 2 2 2 1a b c   . Chứng minh rằng: 
  22 2 2 3 .1 1 1 4
a b c
a a b b c c
b c a
    
  
 Lời giải: Áp dụng bất đẳng thức Cauchy=Schwarz, ta có: 
 
 
2
3
2 2 2 2 2 2 21cyc cyc
cyc
a a b b c ca a
b a b a a b a
 
 
  
   
 2
2 21
cyc
a a b b c c
a b
 

 
 
 
2
22 2 211 .
3
a a b b c c
a b c
 

  
 18 
 2
1
1
3
a a b b c c 


  24
3
a a b b c c   
 Suy ra bất đẳng thức cần chứng minh. 
 Dấu bằng xảy ra khi và chỉ khi
1
3
a b c   . 
*) Các bạn hãy luyện tập thành thạo bất đẳng thức này với một số bài tập nho nhỏ sau: 
 1)Cho . Chứng minh rằng: 
 2) Chứng minh rằng: 
 3) Chứng minh rằng: 
 4) Cho . Chứng minh rằng: 
 5) Cho . Chứng minh rằng: 
 6) Cho . Chứng minh rằng: 
 7) Cho . Tìm min, max của 
 8) Cho . Chứng minh rằng: 
 19 
 9) Tìm min, max của 
 10)Cho . tìm max min của 
V) Bất đẳng thức holder 
 Bất đẳng thức Holder là một bất đẳng thức khá mạnh và có nhiều ứng dụng, 
nhưng rất tiếc nó không được phổ biến ở phổ thông hiện nay. Đặc biệt, bất đẳng thức 
holder còn được coi là dạng tổng quát của bất đẳng thức AM-GM (với m = 2 ) và cách 
chứng minh bất đẳng thức này có sử dụng đến bất đẳng thức AM-GM. Vì thế, chúng tôi 
xin giới thiệu bất đẳng thức này trong chuyên đề. 
 Đầu tiên, xin được trình bày dạng tổng quát của bất đẳng thức : 
Với m dãy số dương ( a1;1 ; a1;2 ;  ; a1;n ) ; ( a2;1 ; a2;2 ;  ; a2;n );.. ; ( am;1 ; am;2 ;  ; 
am;n ) ta có: 
 ; ;
1 11 1
m
m mn n
m
i j i j
j ji i
a a
  
  
        
   
 Đẳng thức xảy ra khi m dãy số tương ứng tỉ lệ. 
 *) Chứng minh: 
 -Sử dụng bất đẳng thức AM-GM, ta có: 
1;1 2;1 ;11;1 2;1 ;1
1; 2; ;
1; 2; ;1 1 1
1 1 1
. . ...
(1) ....
...
m
mm
n n n
n n n
j j m j m j j m jj j j
j j j
m a a aa a a
a a a a a a
  
  
   
    
    
    
     
1;2 2;2 ;21;2 2;2 ;2
1; 2; ;
;1 1 1
11
. . ...
(2) ....
m
mm
n n n
m n
j j m j m i jj j j
ji
m a a aa a a
a a a a
  

   
 
 
 
   
 .. 
 20 
1; 2; ;1; 2; ;
1; 2; ;
;1 1 1
11
. . ...
( ) ....
m
n n m nn n m n
n n n
m n
j j m j m i jj j j
ji
m a a aa a a
n
a a a a
  

   
 
 
 
   
 Cộng từng vế với vế của (1);(2);;(n), ta được : 
;
1 1
; ;
1 11 1
;
11
.
mn
m
i j m mn n
j i
mm i j i j
m n j ji i
m i j
ji
m a
m a a
a
 
  

 
   
   
 
 
 
  

 ; ;
1 11 1
m
m mn n
m
i j i j
j ji i
a a
  
  
         
   
Bất đẳng thức được chứng minh. 
Từ bất đẳng thức holder này, ta có một số hệ quả khá quan trọng sau: 
Hệ quả 1. 
Với a, b, c, x, y, z, m, n, p, là các số thực dương, khi đó, ta có: 
     33 3 3 3 3 3 3 3 3 axa b c x y z m n p m byn czp         
Chứng minh. 
 Thực ra đây chính là hệ quả trực tiếp của bất đẳng thức Holder với m = n = 3. 
Sử dụng bất đẳng thức Holder, ta có: 
3 3 3
3 3 3 3 3 3 3 3 3
a x m
a b c x y z m n p
 
      
   3 3 3 3 3 3 3 3 33
3axm
a b c x y z m n p

     
 Xây dựng tương tự 2 bất đẳng thức (b; y; n) và (c; z; p) rồi cộng theo từng vế, ta 
có điều phải chứng minh. 
 21 
Hệ quả 2. 
Với dãy số a1, a2, a3,, an ta luôn có: 
       1 2 1 21 1 ... 1 1 ...
n
n
n na a a a a a     
Chứng minh. 
Sử dụng bất đẳng thức AM-GM, ta có: 
     1 2 1 2
1 1 1
...
1 1 1 1 1 ... 1nn n
n
a a a a a a
   
     
     
1 21 2
1 2 1 2
...
...
1 1 1 1 1 ... 1
n
nn
n
n n
n a a aaa a
a a a a a a
   
     
Cộng từng vế 2 bất đẳng thức trên ta có điều phải chứng minh. 
Đây là 2 hệ quả khá quan trọng và sẽ dung nhiều trong công cuộc chinh phục đỉnh 
cao bất đẳng thức, vì vậy chúng ta cần nhớ kĩ và vận dung cho thật linh hoạt. 
Do chuyên đề tập trung chủ yếu vào hiệu quả của AM-GM nên chúng ta nên dừng 
các vấn đề về bất đẳng thức holder tại đây. 
Tiếp theo là một số ví dụ có sử dụng bất đẳng thức AM-GM này. 
VI) Ứng dụng của bất đẳng thức AM-GM: 
Bài 1. Cho a, b, c, là các số thực dương. Chứng minh rằng: 
 
2
2 2 2
1 1 1
4
b c c a a b
ab bc ca
a b c a b c
              
   
Lời giải: 
Cách 1: Chúng ta có thể viết lại bất đẳng thức cần chứng minh như sau: 
    
2
2 2 2 2 2 24
cyc
ab a b ab bc ca a b b c c a
 
      
 
 
Giả sử a b c  . Khi đó, áp dụng bất đẳng thức AM-GM, ta có: 
   2 2 2 2 2 24VP ab bc ca a b b c c a     
 22 
    
 
2 2 2 2 2 2 2
2
16
4
a b ab bc ca a b b c c a
a b
    

 
     
 
22 2 2 2 2 2 2
2
4
4
a b ab bc ca a b b c c a
a b
       

Bây giờ, ta chỉ cần chứng minh: 
      2 2 2ab a b cb c b ac a c     
     2 2 2 2 2 2 24a b ab bc ca a b b c c a
a b
     


     2 2 22 2 2ab a b c a b c a b      
   
 2 2 2 2 2 24 a b b c c a
a b ab bc ca
a b
 
    

     
 2 2 22 222 442 2 c a ba bab a b c a b c a b
a b a b

       
 
 
 2 22 24 a bab
ab a b c a b a b
a b a b
                 
     
2 2
2 2ab a b c a b
c a b
a b a b
 
   
 
22ab c
c
a b a b
 
 
Điều này hiển nhiên đúng bởi vì: 
2 22 2c c ab
c c
a b c c a b
   
  
Bất đẳng thức đã được chứng minh. 
Dấu “=” xảy ra khi: a=b=c 
Cách 2. Tương tự như cách 1, bất đẳng thức cần chứng minh tương đương : 
 23 
    
2
2 2 2 2 2 24
cyc
ab a b ab bc ca a b b c c a
 
      
 
 
Ta hãy chú ý đến hằng đẳng thức sau: 
         2 2 22 2 2 2 2 2x y z m n p my nx nz py px mz          
Bây giờ, ta sẽ áp dụng đẳng thức này với: 
     , , , , ,x ab y bc z ca m a b ab n b c bc p a c ac        
Chúng ta có được: 
       
2
2 2
cyc cyc cyc
ab bc ca ab a b ab a b abc c a b
   
         
   
   
Hơn nữa, ta lại có: 
     2 22 2 2 2 2 24
cyc cyc
ab a b a b b c c a ab a b       
Vì vậy, ta có thể viết lại bất đẳng thức cần chứng minh như sau: 
     2 2 2 2 2 2 24
cyc
ab bc ca ab a b a b b c c a
 
       
 
 
      
2
2
cyc cyc
ab bc ca ab a b ab a b
   
       
   
  
     2 2
cyc cyc
ab bc ca ab a b abc c a b       
    2 22 2 0
cyc cyc
a b a b abc a b c a b        
    22 2 2 0
cyc cyc
a b a b abc a a b a c       
Điều này hiển nhiên đúng theo bất đẳng thức Schur bậc 3. 
Bài 2. Cho a, b, c, là các số thực dương. Chứng minh rằng: 
 24 
   
2
1
a b c abc
a b b c c a a b b c c a
   
      
Lời giải: 
Bất đẳng thức đã cho tương đương với: 
     
2
2 2
cyc cyc
a ab
a b b ca b
 
 
  
        
2 2
1 2
abc abc
a b b c c a a b b c c a
 
      
Sử dụng bất đẳng thức quen thuộc sau: 
        
2
2
2
1
cyc
a abc
a b b c c aa b
 
  
 
Ta chỉ cần chứng minh: 
              
2 2
2
cyc
ab abc abc
a b b c a b b c c a a b b c c a
 
        
   2 2 2 2a b b c c a abc abc a b b c c a        
Bây giờ, ta giả sử  min , ,c a b c , áp dụng bất đẳng thức AM-GM, ta có: 
   2 2 2 2a b b c c a abc a ab c bc a b       
       
2 2
a a b b c a a c b c
bc a b
   
    
           2 .
2 2
a a b b c a a b b c
bc a b bc a b
   
     
     2abc a b b c c a    
Bất đẳng thức được chứng minh. Dấu “=” xảy ra khi: a = b = c. 
 25 
Bài 3. Cho a, b, c, là các số thực dương. Chứng minh rằng: 
 
 
 
 
 
 
2 2 2
3
2 2 2
b c a c a b
a b c a b a c b c a b c
  
  
      
Lời giải: 
Cách 1. Sau khi sử dụng bất đẳng thức AM-GM, ta có được bất đẳng thức cần chứng 
minh tương đương 

File đính kèm:

  • pdfBDT_AM-GM.pdf
Bài giảng liên quan